10
$\begingroup$

Consider a generic $n \times n$ matrix $M$.

Define the $(n-1) \times n$ matrix $M_q$ to be $M$ with the $q$th row omitted, and assume that $M_q$ possesses a right inverse, $R_q$:

$$R_q = M_q^T (M_q M_q^T)^{-1}$$

The components of $R_q$ will be rational functions of the components of $M$, with the determinant $\det(M_q M_q^T)$ as their denominator.

Now, consider the $n \times n$ matrix $C$ of cofactors of $M$. That is, $C_{i j}$ is $(-1)^{i+j}$ times the determinant of the submatrix of $M$ obtained by omitting the $i$th row and the $j$th column.

I have found by direct computation for low values of $n$ that the antisymmetric products of the $q$th row of $C$ and the components of $R_q$:

$$(A_q)_{i j k} = C_{q i} (R_q)_{j k} - C_{q j} (R_q)_{i k}$$

are polynomials of degree $n-2$ in the components of $M$, with the determinant that one might expect to be present in the denominator, inherited from $R_q$, factoring out.

To give an example, for $n=3$:

$$M_1= \left( \begin{array}{ccc} m_{2,1} & m_{2,2} & m_{2,3} \\ m_{3,1} & m_{3,2} & m_{3,3} \\ \end{array} \right)$$

$$C= \left( \begin{array}{ccc} m_{2,2} m_{3,3}-m_{2,3} m_{3,2} & m_{2,3} m_{3,1}-m_{2,1} m_{3,3} & m_{2,1} m_{3,2}-m_{2,2} m_{3,1} \\ m_{1,3} m_{3,2}-m_{1,2} m_{3,3} & m_{1,1} m_{3,3}-m_{1,3} m_{3,1} & m_{1,2} m_{3,1}-m_{1,1} m_{3,2} \\ m_{1,2} m_{2,3}-m_{1,3} m_{2,2} & m_{1,3} m_{2,1}-m_{1,1} m_{2,3} & m_{1,1} m_{2,2}-m_{1,2} m_{2,1} \\ \end{array} \right)$$

$$ R_1 = M_1^T (M_1 M_1^T)^{-1} = \\ {\scriptsize \frac{ \left( \begin{array}{cc} m_{2,1} \left(m_{3,2}^2+m_{3,3}^2\right)-m_{3,1} \left(m_{2,2} m_{3,2}+m_{2,3} m_{3,3}\right) & m_{3,1} \left(m_{2,2}^2+m_{2,3}^2\right) -m_{2,1} \left(m_{2,2} m_{3,2}+m_{2,3} m_{3,3}\right) \\ m_{2,2} \left(m_{3,1}^2+m_{3,3}^2\right)-m_{3,2} \left(m_{2,1} m_{3,1}+m_{2,3} m_{3,3}\right) & m_{3,2} \left(m_{2,1}^2+m_{2,3}^2\right) -m_{2,2} \left(m_{2,1} m_{3,1}+m_{2,3} m_{3,3}\right) \\ m_{2,3} \left(m_{3,1}^2+m_{3,2}^2\right)-m_{3,3} \left(m_{2,1} m_{3,1}+m_{2,2} m_{3,2}\right) & m_{3,3} \left(m_{2,1}^2+m_{2,2}^2\right) -m_{2,3} \left(m_{2,1} m_{3,1}+m_{2,2} m_{3,2}\right) \\ \end{array} \right)} {\left(m_{3,2}^2+m_{3,3}^2\right) m_{2,1}^2-2 m_{2,3} m_{3,1} m_{3,3} m_{2,1}+m_{2,3}^2 \left(m_{3,1}^2+m_{3,2}^2\right)-2 m_{2,2} m_{3,2} \left(m_{2,1} m_{3,1}+m_{2,3} m_{3,3}\right)+m_{2,2}^2 \left(m_{3,1}^2+m_{3,3}^2\right)} } $$

$$(A_1)_{i j k}= C_{1 i} (R_1)_{j k} - C_{1 j} (R_1)_{i k} = \left( \begin{array}{ccc} \left( \begin{array}{c} 0 \\ 0 \\ \end{array} \right) & \left( \begin{array}{c} m_{3,3} \\ -m_{2,3} \\ \end{array} \right) & \left( \begin{array}{c} -m_{3,2} \\ m_{2,2} \\ \end{array} \right) \\ \left( \begin{array}{c} -m_{3,3} \\ m_{2,3} \\ \end{array} \right) & \left( \begin{array}{c} 0 \\ 0 \\ \end{array} \right) & \left( \begin{array}{c} m_{3,1} \\ -m_{2,1} \\ \end{array} \right) \\ \left( \begin{array}{c} m_{3,2} \\ -m_{2,2} \\ \end{array} \right) & \left( \begin{array}{c} -m_{3,1} \\ m_{2,1} \\ \end{array} \right) & \left( \begin{array}{c} 0 \\ 0 \\ \end{array} \right) \\ \end{array} \right)$$

My question is: why are the components of $A_q$ polynomials rather than rational functions? Can this be proved for general $n$? And can $A_q$ be reduced, for general $n$, to a simpler expression that makes no reference to $R_q$?

(Edited to change question from sum to individual components).

$\endgroup$
2
  • 2
    $\begingroup$ Am I being stupid or is the thing polynomial even if you don't sum over $k$ ? And (without summing) it should be an $\left(n-2\right)\times\left(n-2\right)$-minor of $M$ (up to sign). (I am not 100% sure of my proof so far.) $\endgroup$ Jul 28, 2014 at 13:44
  • $\begingroup$ You're absolutely right, there's no need to sum. The summation arose in the problem I was tackling and I assumed it was crucial. I should have checked! The separate results do indeed look like higher-order minors (though it's not yet clear to me why). $\endgroup$
    – Greg Egan
    Jul 28, 2014 at 14:07

1 Answer 1

8
$\begingroup$

$\let\sumnonlimits\sum \let\prodnonlimits\prod \renewcommand{\sum}{\sumnonlimits\limits} \renewcommand{\prod}{\prodnonlimits\limits} $ I will write $R$ for $R_{q}$, because $q$ is constant. In the following, I am going to assume that $R$ is an arbitrary right inverse of $M_{q}$, rather than the specific right inverse $M_{q}^{T}\left( M_{q}M_{q}^{T}\right) ^{-1}$ which you have suggested. This makes the statement a tad more general and rids us of a red herring.

I shall prove that for any $i\in\left\{ 1,2,...,n\right\} $, $j\in\left\{ 1,2,...,n\right\} $ and $k\in\left\{ 1,2,...,n-1\right\} $, the value $C_{qi}R_{jk}-C_{qj}R_{ik}$ is a homogeneous polynomial of degree $n-2$ in the entries of the matrix $M_{q}$ (independently, and independent, of the choice of right inverse $R$).

For every $\ell\in\left\{ 1,2,...,n\right\} $, we abbreviate $C_{q\ell}$ by $v_{q}$. Thus,

$v_{\ell}=C_{q\ell}=\left( -1\right) ^{q+\ell}\det\left( \underbrace{M\text{ without row }q}_{=M_{q}}\text{ and column }\ell\right) $

(1) $=\left( -1\right) ^{q+\ell}\det\left( M_{q}\text{ without column }\ell\right) $.

From this it easy to obtain

(2) $\sum_{\ell\in\left\{ 1,2,...,n\right\} }\left( M_{q}\right) _{j\ell}v_{\ell}=0$ for every $j \in \left\{1,2,...,n-1\right\}$.

[Proof of (2): Let $j \in \left\{1,2,...,n-1\right\}$. Let $G$ be the result of inserting a copy of row $j$ of the matrix $M_{q}$ between the rows $q-1$ and $q$ of this matrix. Then, $G$ is an $n\times n$-matrix with two equal rows, and hence has determinant $\det G=0$. But Laplace expansion of $\det G$ along the row we have inserted yields

$\det G=\sum_{\ell\in\left\{ 1,2,...,n\right\} }\left( M_{q}\right) _{j\ell}\left( -1\right) ^{q+\ell}\det\left( M_{q}\text{ without column }\ell\right) $,

since the entries of this row are $\left( M_{q}\right) _{j\ell}$ and the cofactors corresponding to this row are $\left( -1\right) ^{q+\ell} \det\left( M_{q}\text{ without column }\ell\right) $. Now, (1) yields

$\sum_{\ell\in\left\{ 1,2,...,n\right\} }\left( M_{q}\right) _{j\ell }v_{\ell}$

$=\sum_{\ell\in\left\{ 1,2,...,n\right\} }\left( M_{q}\right) _{j\ell }\left( -1\right) ^{q+\ell}\det\left( M_{q}\text{ without column } \ell\right) $

$=\det G=0$,

so that (2) is proven.]

Since $M$ is generic, its cofactors $v_{1}$, $v_{2}$, $...$, $v_{n}$ are nonzero.

Fix $i\in\left\{ 1,2,...,n\right\} $. Let $N_{i}$ denote the matrix $M_{q}$ without column $i$. Thus, $\det\left( N_{i}\right) =\det\left( M_{q}\text{ without column }i\right) =\left( -1\right) ^{q+i}v_{i}$ (because (1) yields $v_{i}=\left( -1\right) ^{q+i}\det\left( M_{q}\text{ without column }i\right) $).

Let $S_{i}$ denote the matrix $\left( R_{jk}-\dfrac{v_{j}}{v_{i}} R_{ik}\right) _{j\in\left\{ 1,2,...,n\right\} ;\ k\in\left\{ 1,2,...,n-1\right\} }$. The $i$-th row of this matrix $S_{i}$ is zero; let $S_{i}^{\prime}$ denote the matrix $S_{i}$ without row $i$.

Now, we claim that $N_{i}S_{i}^{\prime}=I_{n-1}$ (the $\left( n-1\right) \times\left( n-1\right) $ identity matrix). Indeed, for every $\left( j,k\right) \in\left\{ 1,2,...,n-1\right\} ^{2}$, the $\left( j,k\right) $-th entry of $N_{i}S_{i}^{\prime}$ is

$\left( N_{i}S_{i}^{\prime}\right) _{jk}=\sum_{u\in\left\{ 1,2,...,n-1\right\} }\left( N_{i}\right) _{ju}\left( S_{i}^{\prime }\right) _{uk}$

$=\sum_{\ell\in\left\{ 1,2,...,n\right\} \setminus\left\{ i\right\} }\left( M_{q}\right) _{j\ell}\underbrace{\left( S_{i}\right) _{\ell k} }_{=R_{\ell k}-\dfrac{v_{\ell}}{v_{i}}R_{ik}}$

(since $N_{i}$ is the matrix $M_{q}$ without column $i$, while $S_{i}^{\prime }$ is the matrix $S_{i}$ without row $i$)

$=\sum_{\ell\in\left\{ 1,2,...,n\right\} \setminus\left\{ i\right\} }\left( M_{q}\right) _{j\ell}\left( R_{\ell k}-\dfrac{v_{\ell}}{v_{i} }R_{ik}\right) $

$=\sum_{\ell\in\left\{ 1,2,...,n\right\} }\left( M_{q}\right) _{j\ell }\left( R_{\ell k}-\dfrac{v_{\ell}}{v_{i}}R_{ik}\right) $

(here, we added an $\ell=i$ addend to the sum; this did not change the sum because this addend is $0$)

$=\underbrace{\sum_{\ell\in\left\{ 1,2,...,n\right\} }\left( M_{q}\right) _{j\ell}R_{\ell k}}_{\substack{=\left( M_{q}R\right) _{jk}=\delta _{jk}\\\text{(since }R\text{ is a}\\\text{right inverse of }M_{q}\text{)} }}-\underbrace{\sum_{\ell\in\left\{ 1,2,...,n\right\} }\left( M_{q}\right) _{j\ell}\dfrac{v_{\ell}}{v_{i}}R_{ik}}_{=\dfrac{1}{v_{i}}R_{ik}\sum_{\ell \in\left\{ 1,2,...,n\right\} }\left( M_{q}\right) _{j\ell}v_{\ell}}$

$=\delta_{jk}-\dfrac{1}{v_{i}}R_{ik}\underbrace{\sum_{\ell\in\left\{ 1,2,...,n\right\} }\left( M_{q}\right) _{j\ell}v_{\ell}} _{\substack{=0\\\text{(by \textbf{(2)})}}}=\delta_{jk}$.

This shows that $N_{i}S_{i}^{\prime}=I_{n-1}$, and thus $S_{i}^{\prime} =N_{i}^{-1}$ (since $N_{i}$ and $S_{i}^{\prime}$ are $\left( n-1\right) \times\left( n-1\right) $-matrices).

Now, we recall Cramer's rule for the inverse of a matrix. It essentially says that the inverse of a square matrix is obtained by dividing its adjoint by its determinant. In other words, if $X$ is an invertible $p\times p$-matrix for some $p\in\mathbb{N}$, and $j$ and $k$ are elements of $\left\{ 1,2,...,p\right\} $, then

(3) $\left( X^{-1}\right) _{jk}=\dfrac{1}{\det X}\left( -1\right) ^{j+k}\det\left( X\text{ without row }k\text{ and column }j\right) $.

Now, recall that $S_{i}^{\prime}=N_{i}^{-1}$. Hence,

$\left( S_{i}^{\prime}\right) _{jk}=\left( N_{i}^{-1}\right) _{jk} =\dfrac{1}{\det\left( N_{i}\right) }\left( -1\right) ^{j+k}\det\left( N_{i}\text{ without row }k\text{ and column }j\right) $ (by (3))

$=\dfrac{1}{\left( -1\right) ^{q+i}v_{i}}\left( -1\right) ^{j+k} \det\left( N_{i}\text{ without row }k\text{ and column }j\right) $ (since $\det\left( N_{i}\right) =\left( -1\right) ^{q+i}v_{i}$)

$=\dfrac{1}{v_{i}}\left( -1\right) ^{i+j+q+k}\det\left( N_{i}\text{ without row }k\text{ and column }j\right) $

for all $\left( j,k\right) \in\left\{ 1,2,...,n-1\right\} ^{2}$. In other words,

(4) $v_{i}\left( S_{i}^{\prime}\right) _{jk}=\left( -1\right) ^{i+j+q+k}\det\left( N_{i}\text{ without row }k\text{ and column }j\right) $

for all $\left( j,k\right) \in\left\{ 1,2,...,n-1\right\} ^{2}$.

Let $\mathbf{B}$ be the (unique) increasing bijection from $\left\{ 1,2,...,n\right\} \setminus\left\{ i\right\} $ to $\left\{ 1,2,...,n-1\right\} $. Then, for all $j\in\left\{ 1,2,...,n\right\} \setminus\left\{ i\right\} $ and $k\in\left\{ 1,2,...,n-1\right\} $, we have

(5) $v_{i}\left( S_{i}\right) _{jk}=\left( -1\right) ^{i+\mathbf{B} \left( j\right) +q+k}\det\left( M_{q}\text{ without row }k\text{ and columns }i\text{ and }j\right) $.

(Indeed, this follows from (4), applied to $\mathbf{B}\left( j\right) $ instead of $j$, because $\left( S_{i}^{\prime}\right) _{\mathbf{B}\left( j\right) ,k}=\left( S_{i}\right) _{jk}$ (since $S_{i}^{\prime}$ is the matrix $S_{i}$ without row $i$) and because column $j$ of $M_{q}$ is column $\mathbf{B}\left( j\right) $ of $N_{i}$ (since $N_{i}$ is the matrix $M_{q}$ without column $i$).)

Now, fix $j\in\left\{ 1,2,...,n\right\} $ and $k\in\left\{ 1,2,...,n-1\right\} $. We need to show that $C_{qi}R_{jk}-C_{qj}R_{ik}$ is a homogeneous polynomial of degree $n-2$ in the entries of the matrix $M_{q}$. We WLOG assume that $j\neq i$ (since otherwise, $C_{qi}R_{jk}-C_{qj}R_{ik} =0$). Thus, $j\in\left\{ 1,2,...,n\right\} \setminus\left\{ i\right\} $. Since $C_{qi}=v_{i}$ and $C_{qj}=v_{j}$ (by the definitions of $v_{i}$ and $v_{j}$), we have

$\underbrace{C_{qi}}_{=v_{i}}R_{jk}-\underbrace{C_{qj}}_{=v_{j}}R_{ik} =v_{i}R_{jk}-v_{j}R_{ik}=v_{i}\underbrace{\left( R_{jk}-\dfrac{v_{j}}{v_{i} }R_{ik}\right) }_{\substack{=\left( S_{i}\right) _{jk}\\\text{(by the definition of }S_{i}\text{)}}}$

(6) $=v_{i}\left( S_{i}\right) _{jk}=\left( -1\right) ^{i+\mathbf{B}\left( j\right) +q+k}\det\left( M_{q}\text{ without row }k\text{ and columns }i\text{ and }j\right) $

(by (5)),

which is obviously a homogeneous polynomial of degree $n-2$ in the entries of the matrix $M_{q}$ (and independent of the choice of $R$), qed.

Thanks for a very nice question, and sorry for this mess of an answer...

PS. I believe the genericity of $M$ is not required for (6) to hold. Does anyone see a nice proof of this? I don't.

PPS. Here is a more general statement which, I think, is true. Let $A$ be a commutative ring. Let $N$ be an $\left(n-1\right) \times n$-matrix over $A$. Let $s \in A^n$ be a vector. For every $\ell \in \left\{1,2,...,n\right\}$, let $p_\ell$ denote the scalar $\left( -1\right) ^{\ell}\det\left( N \text{ without column }\ell\right)$. If $w$ is a vector and $i$ is an integer, we denote by $w_i$ the $i$-th coordinate of $w$ (whenever this makes sense). Then, every two distinct $i \in \left\{1,2,...,n\right\}$ and $j \in \left\{1,2,...,n\right\}$ satisfy

(11) $p_i s_j - p_j s_i = \sum_{k=1}^{n-1} \pm \left(Ns\right)_k \det\left(N \text{ without row } k \text{ and columns } i \text{ and } j\right)$,

where $\pm$ is something like $\left(-1\right)^{i+j+\left[i<j\right]+k}$ using the Iverson bracket.

If this is proven (and this shouldn't be too hard -- it's a polynomial identity, so you can assume as much genericity as you wish), the original result is obtained by setting $N = M_q$ and $s = \left(k\text{-th column of } R\right)$.

PPPS. Indeed, (11) is not hard to prove. Let $e_1, e_2, ..., e_n$ be the $n$ standard basis vectors of $A^n$. Let $P$ be the $n \times \left(n-1\right)$-matrix whose columns (from left to right) are $e_1, e_2, ..., \widehat{e_i}, ..., \widehat{e_j}, ..., e_n, s$ (where $\widehat{\text{something}}$ means omission, and the order of $i$ and $j$ is not necessarily the one we have shown). Then, $NP$ is the $\left(n-1\right)\times \left(n-1\right)$-matrix whose columns (from left to right) are the columns $1, 2, ..., \widehat{i}, ..., \widehat{j}, ..., n$ of $N$ and the column-vector $Ns$. The right hand side of (11) is the determinant of this matrix $NP$, computed by Laplace expansion along its last column. The left hand side of (11) is the same determinant, computed using the Cauchy-Binet formula (which takes a rather simple form here because if we remove the $\ell$-th row from $P$ for some $\ell \notin \left\{i, j\right\}$, then the resulting matrix has two rows with only their rightmost entries nonzero, and so has determinant $0$).

PPPPS. Here is a different way to formulate the above proof of (11), using exterior algebra instead of the Cauchy-Binet formula and giving some more detail.

Let $e_{1}$, $e_{2}$, $...$, $e_{n}$ be the $n$ standard basis vectors of $A^{n}$. Let $f_{1}$, $f_{2}$, $...$, $f_{n-1}$ be the $n-1$ standard basis vectors of $A^{n-1}$. We identify the matrix $N\in A^{\left( n-1\right) \times n}$ with the $A$-linear map $A^{n}\rightarrow A^{n-1}$ it represents. As usual, we use the notation ''$\widehat{\text{some term}}$'' for omission of a term in a product or list (for example, $\left( 1,2,...,\widehat{5},...,8\right) =\left( 1,2,3,4,6,7,8\right) $). We also use the Iverson bracket notation, i.e., whenever $\mathcal{A}$ is a logical statement, we write $\left[ \mathcal{A}\right] $ for the integer $\left\{ \begin{array} [c]{c} 1,\text{ if }\mathcal{A}\text{ is true;}\\ 0,\text{ if }\mathcal{A}\text{ is false} \end{array} \right. $.

Let $\mathbf{f}$ be the element $f_{1}\wedge f_{2}\wedge...\wedge f_{n-1}$ of $\wedge^{n-1}\left( A^{n-1}\right) $. It is known that $\left( \mathbf{f}\right) $ is an $A$-module basis of $\wedge^{n-1}\left( A^{n-1}\right) $.

Let $i$ and $j$ be two distinct elements of $\left\{ 1,2,...,n\right\} $. We have $s=\sum_{k=1}^{n}s_{k}e_{k}$, so that

$e_{1}\wedge e_{2}\wedge...\wedge\widehat{e_{i}}\wedge...\wedge\widehat{e_{j} }\wedge...\wedge e_{n}\wedge s$

$=e_{1}\wedge e_{2}\wedge...\wedge\widehat{e_{i}}\wedge...\wedge \widehat{e_{j}}\wedge...\wedge e_{n}\wedge\left( \sum_{k=1}^{n}s_{k} e_{k}\right) $

(12) $=\sum_{k=1}^{n}s_{k}e_{1}\wedge e_{2}\wedge...\wedge\widehat{e_{i} }\wedge...\wedge\widehat{e_{j}}\wedge...\wedge e_{n}\wedge e_{k}$

(where the notation $e_{1}\wedge e_{2}\wedge...\wedge\widehat{e_{i}} \wedge...\wedge\widehat{e_{j}}\wedge...\wedge e_{n}$ means that both $e_{i}$ and $e_{j}$ are omitted, but does not necessarily imply that $i<j$). Of all the addends of the sum on the right hand side of (12), only those for $k=i$ and for $k=j$ have a chance to be nonzero (every other summand contains a wedge product with two equal factors, and thus vanishes). Hence, (12) simplifies to

$e_{1}\wedge e_{2}\wedge...\wedge\widehat{e_{i}}\wedge...\wedge\widehat{e_{j} }\wedge...\wedge e_{n}\wedge s$

$=s_{i}\underbrace{e_{1}\wedge e_{2}\wedge...\wedge\widehat{e_{i}} \wedge...\wedge\widehat{e_{j}}\wedge...\wedge e_{n}\wedge e_{i}}_{=\left( -1\right) ^{i+\left[ i<j\right] }e_{1}\wedge e_{2}\wedge...\wedge \widehat{e_{j}}\wedge...\wedge e_{n}}+s_{j}\underbrace{e_{1}\wedge e_{2} \wedge...\wedge\widehat{e_{i}}\wedge...\wedge\widehat{e_{j}}\wedge...\wedge e_{n}\wedge e_{j}}_{=\left( -1\right) ^{j+1-\left[ i<j\right] }e_{1}\wedge e_{2}\wedge...\wedge\widehat{e_{i}}\wedge...\wedge e_{n}}$

$=s_{i}\left( -1\right) ^{i+\left[ i<j\right] }e_{1}\wedge e_{2} \wedge...\wedge\widehat{e_{j}}\wedge...\wedge e_{n}+s_{j}\left( -1\right) ^{j+1-\left[ i<j\right] }e_{1}\wedge e_{2}\wedge...\wedge\widehat{e_{i} }\wedge...\wedge e_{n}$

$=\left( -1\right) ^{\left[ i<j\right] +i+j-1}\left( s_{j}\left( -1\right) ^{i}e_{1}\wedge e_{2}\wedge...\wedge\widehat{e_{i}}\wedge...\wedge e_{n}-s_{i}\left( -1\right) ^{j}e_{1}\wedge e_{2}\wedge...\wedge \widehat{e_{j}}\wedge...\wedge e_{n}\right) $.

Applying the linear map $\wedge^{n-1}N$ to both sides of this equality results in

$\left( \wedge^{n-1}N\right) \left( e_{1}\wedge e_{2}\wedge...\wedge \widehat{e_{i}}\wedge...\wedge\widehat{e_{j}}\wedge...\wedge e_{n}\wedge s\right) $

$=\left( -1\right) ^{\left[ i<j\right] +i+j-1}\left( s_{j}\left( -1\right) ^{i}\underbrace{\left( \wedge^{n-1}N\right) \left( e_{1}\wedge e_{2}\wedge...\wedge\widehat{e_{i}}\wedge...\wedge e_{n}\right) } _{=\det\left( N\text{ without column }i\right) \mathbf{f}}\right. $

$\left. -s_{i}\left( -1\right) ^{j}\underbrace{\left( \wedge ^{n-1}N\right) \left( e_{1}\wedge e_{2}\wedge...\wedge\widehat{e_{j}} \wedge...\wedge e_{n}\right) }_{=\det\left( N\text{ without column }j\right) \mathbf{f}}\right) $

$=\left( -1\right) ^{\left[ i<j\right] +i+j-1}\left( s_{j} \underbrace{\left( -1\right) ^{i}\det\left( N\text{ without column }i\right) }_{=p_{i}} \mathbf{f} -s_{i}\underbrace{\left( -1\right) ^{j} \det\left( N\text{ without column }j\right) }_{=p_{j}} \mathbf{f} \right) $

$=\left( -1\right) ^{\left[ i<j\right] +i+j-1}\left( s_{j}p_{i} \mathbf{f}-s_{i}p_{j}\mathbf{f}\right) =\left( -1\right) ^{\left[ i<j\right] +i+j-1}\left( p_{i}s_{j}-p_{j}s_{i}\right) \mathbf{f}$,

so that

$\left( p_{i}s_{j}-p_{j}s_{i}\right) \mathbf{f}$

(13) $=\left( -1\right) ^{\left[ i<j\right] +i+j-1}\left( \wedge^{n-1}N\right) \left( e_{1}\wedge e_{2}\wedge...\wedge\widehat{e_{i} }\wedge...\wedge\widehat{e_{j}}\wedge...\wedge e_{n}\wedge s\right) $.

On the other hand, using $\wedge$ to denote the multiplication in the exterior algebra $\wedge\left( A^{n-1}\right) $, we see

$\left( \wedge^{n-1}N\right) \left( e_{1}\wedge e_{2}\wedge...\wedge \widehat{e_{i}}\wedge...\wedge\widehat{e_{j}}\wedge...\wedge e_{n}\wedge s\right) $

$=\underbrace{\left( \wedge^{n-2}N\right) \left( e_{1}\wedge e_{2} \wedge...\wedge\widehat{e_{i}}\wedge...\wedge\widehat{e_{j}}\wedge...\wedge e_{n}\right) }_{=\sum_{\ell=1}^{n-1}\det\left( N\text{ without columns }i\text{ and }j\text{ and row }\ell\right) f_{1}\wedge f_{2}\wedge ...\wedge\widehat{f_{\ell}}\wedge...\wedge f_{n-1}}\wedge\underbrace{\left( Ns\right) }_{=\sum_{k=1}^{n-1}\left( Ns\right) _{k}f_{k}}$

$=\left( \sum_{\ell=1}^{n-1}\det\left( N\text{ without columns }i\text{ and }j\text{ and row }\ell\right) f_{1}\wedge f_{2}\wedge...\wedge \widehat{f_{\ell}}\wedge...\wedge f_{n-1}\right) \wedge\left( \sum _{k=1}^{n-1}\left( Ns\right) _{k}f_{k}\right) $

(14) $=\sum_{\ell=1}^{n-1}\sum_{k=1}^{n-1}\left( Ns\right) _{k} \det\left( N\text{ without columns }i\text{ and }j\text{ and row } \ell\right) f_{1}\wedge f_{2}\wedge...\wedge\widehat{f_{\ell}}\wedge...\wedge f_{n-1}\wedge f_{k}$.

Of all the addends of the inner sum on the right hand side of (14), only those for $k=\ell$ have a chance to be nonzero (because all other addends have the form $f_{1}\wedge f_{2}\wedge...\wedge\widehat{f_{\ell}}\wedge...\wedge f_{n-1}\wedge f_{k}$ for $k\neq\ell$, which is a wedge product with two equal factors and thus equals $0$). Hence, (14) simplifies to

$\left( \wedge^{n-1}N\right) \left( e_{1}\wedge e_{2}\wedge...\wedge \widehat{e_{i}}\wedge...\wedge\widehat{e_{j}}\wedge...\wedge e_{n}\wedge s\right) $

$=\sum_{\ell=1}^{n-1}\left( Ns\right) _{\ell}\det\left( N\text{ without columns }i\text{ and }j\text{ and row }\ell\right) \underbrace{f_{1}\wedge f_{2}\wedge...\wedge\widehat{f_{\ell}}\wedge...\wedge f_{n-1}\wedge f_{\ell} }_{=\left( -1\right) ^{n-1-\ell}f_{1}\wedge f_{2}\wedge...\wedge f_{n-1}}$

$=\sum_{\ell=1}^{n-1}\left( Ns\right) _{\ell}\det\left( N\text{ without columns }i\text{ and }j\text{ and row }\ell\right) \left( -1\right) ^{n-1-\ell}\underbrace{f_{1}\wedge f_{2}\wedge...\wedge f_{n-1}}_{=\mathbf{f} }$

$=\sum_{\ell=1}^{n-1}\left( Ns\right) _{\ell}\det\left( N\text{ without columns }i\text{ and }j\text{ and row }\ell\right) \left( -1\right) ^{n-1-\ell}\mathbf{f}$.

Thus, (13) becomes

$\left( p_{i}s_{j}-p_{j}s_{i}\right) \mathbf{f}$

$=\left( -1\right) ^{\left[ i<j\right] +i+j-1}\underbrace{\left( \wedge^{n-1}N\right) \left( e_{1}\wedge e_{2}\wedge...\wedge\widehat{e_{i} }\wedge...\wedge\widehat{e_{j}}\wedge...\wedge e_{n}\wedge s\right) } _{=\sum_{\ell=1}^{n-1}\left( Ns\right) _{\ell}\det\left( N\text{ without columns }i\text{ and }j\text{ and row }\ell\right) \left( -1\right) ^{n-1-\ell}\mathbf{f}}$

$=\left( -1\right) ^{\left[ i<j\right] +i+j-1}\sum_{\ell=1}^{n-1}\left( Ns\right) _{\ell}\det\left( N\text{ without columns }i\text{ and }j\text{ and row }\ell\right) \left( -1\right) ^{n-1-\ell}\mathbf{f}$.

Since $\left( \mathbf{f}\right) $ is a basis of $\wedge^{n-1}\left( A^{n-1}\right) $, we can compare coefficients before $\mathbf{f}$ in this equality, and obtain

$p_{i}s_{j}-p_{j}s_{i}$

$=\left( -1\right) ^{\left[ i<j\right] +i+j-1}\sum_{\ell=1}^{n-1}\left( Ns\right) _{\ell}\det\left( N\text{ without columns }i\text{ and }j\text{ and row }\ell\right) \left( -1\right) ^{n-1-\ell}$

$=\sum_{\ell=1}^{n-1}\left( -1\right) ^{\left[ i<j\right] +i+j+n-\ell }\left( Ns\right) _{\ell}\det\left( N\text{ without columns }i\text{ and }j\text{ and row }\ell\right) $

$=\sum_{k=1}^{n-1}\left( -1\right) ^{\left[ i<j\right] +i+j+n-k}\left( Ns\right) _{k}\det\left( N\text{ without columns }i\text{ and }j\text{ and row }k\right) $.

This proves (11), up to all the sign errors I surely have made.

As a consequence, we obtain a new proof of your statement, and it does no longer rely on genericity of $M$.

$\endgroup$
4
  • $\begingroup$ Thanks! It might take me some time to work through this and be sure I understand it. $\endgroup$
    – Greg Egan
    Jul 28, 2014 at 15:16
  • $\begingroup$ You're welcome! Let me know if something is wrong. $\endgroup$ Jul 28, 2014 at 15:24
  • $\begingroup$ This looks perfect to me. I'll have to think about non-generic $M$; so long as $M_q$ still has a right inverse maybe we can argue by continuity from the generic case. $\endgroup$
    – Greg Egan
    Jul 28, 2014 at 23:51
  • $\begingroup$ That's a wonderful generalisation! I had tried to use the Cauchy-Binet formula on the original problem, but I couldn't see how to make it fit. $\endgroup$
    – Greg Egan
    Jul 29, 2014 at 23:13

Your Answer

By clicking “Post Your Answer”, you agree to our terms of service and acknowledge you have read our privacy policy.

Not the answer you're looking for? Browse other questions tagged or ask your own question.